PHYSICS QUESTIONS

अब Quizwiz के साथ अपने होमवर्क और परीक्षाओं को एस करें!

The glass that is used as a beam splitter is replaced with glass that is identical except that it has a 10% higher index of refraction. Which of the following changes will occur to the pinhole image? A. It will move. B. It will become larger. C. It will become smaller. D. It will become more clear.

A is correct A refraction shifts the rays of light over a bit. If the index of refraction is increased, this increases the shift. This will cause a shift in the position of the light striking the retroflector.

What is considered a state function? A. Entropy B. Work C. Acceleration D. Heat

A is correct. State functions are represented by internal energy, enthalpy, and entropy. These are quantities that can be described quantitatively such as an equilibrium state of a thermodynamic system. They DO NOT depend on the path taken.

endergonic reaction

A non-spontaneous chemical reaction in which free energy is absorbed from the surroundings.

exergonic reaction

A spontaneous chemical reaction in which there is a net release of free energy.

The explanation for the fact that radioactive isotopes of an element exhibit the same chemical behavior as the stable isotopes of the element is that each has the same: A. atomic number. B. number of neutrons. C. mass number. D. atomic weight.

A. Atomic Number

What would be the mathematical expression for h (maximum height) of a projectile in motion? A. mv^2/2. B. v^2/(2g). C. mg. D. mv.

At max h (PE=KE) so mgh = 1/2 mv^2 h = 1/2 v^2 / g

Changing which of the following will change the focal length of the convex mirror? A. Index of refraction of the mirror B. Radius of curvature of the mirror C. Position of the lens at B D. Focal length of the lens at B

B is the correct answer The focal length of the mirror depends only on the radius of the curvature. Because there is no refraction, the index of refraction is irrelevant

What is the work done by the patient during a 3-minute exercise on the bicycle when P=30 W? A.5400 J B.90 J C.6 J D.0.16 J

The answer is A W = Fd W = Pt W = (30W)(3 min)(60 sec) = 5400 J

Given two isomers, one with the largest substituent in the front and one with the substituent in the back, which one is more stable?

The one that is thermodynamically favored, with the largest substituent in the front, will be more stable.

A circuit is comprised of a 12-volt battery with three light bulbs with the same resistance. If two of the light bulbs are wired in parallel and are in series with the third light bulb, what happens when one of the parallel light bulbs is unscrewed? A. All of the light bulbs remain lit. B. Two of the light bulbs remain lit. C. One of the light bulbs remains lit. D. All of the light bulbs go out.

the answer is B

In what way do electric field lines move in the context of their equipotential surfaces?

Field lines move perpendicular to the equipotential surfaces and exit from positive and enter negative charges.

Would a gas behave more ideally under low or high temperatures?

High temps

Venturi effect

If velocity goes up, pressure goes down. Also be sure not to forget how the continuity equation ties into the bernoulli equation. If Area goes down, Velocity goes up.

Resistivity

Inverse of conductivity (1/C)

Calculate the frequency of a beam of light with a wavelength of 6x10^-7 c = 3x10^8

f = c/λ f = 3x10^8/6x10^-7 = 0.5x10^15 (subtract exponents when dividing)

A circuit constructed with a 12V battery and 4 Identical Resistors, each with a resistance of 16 Ohm, hooked up in parallel. What is the total power dissipated? A. 4 J B. 4 W C. 18 J D. 36 W

1/R = 1/16 (4) = 4/16 = 1/4 R = 4 Ohm P = I^2/R = 12^2/4 = 144/4 = 36 W

A 60-kg runner raises his center of mass approximately 0.5 m with each step. Although his leg muscles act as a spring, recapturing the energy each time his feet touch down, there's an average 10% loss with each compression. What must the runner's additional power output be to account for just this loss, if he averages 0.8 s per stride?

37 W The gravitational potential energy at the runner's height is: PE = (60 kg)(10 m/s2)(0.5 m) = 300 J Loses 10% or 30 J P = W/t = (30 J)/(0.8 s) = 40 W

Visible light travels more slowly through an optically dense medium than through a vacuum. A possible explanation for this could be that the light: A. is absorbed and re-emitted by the atomic structure of the optically dense medium. B. is absorbed and re-emitted by the nucleus of the material in the optically dense medium. C. bounces around randomly inside of the optically dense medium before emerging. D. loses amplitude as it passes through the optically dense medium.

A. is correct B is incorrect because the nucleus is involved. C is incorrect because the motion of the photons is certainly not random. D is true but does not answer the question

What is the magnitude of the electric field between the two electrodes in ionization type detectors? Q=5V d= 3 cm A. 1.5 N/C B. 1.66 N/C C. 15 N/C D. 166 N/C

An electric field can be given in volts per meter. This means the electric field value is 5/(0.03) = 166.7 V/m. Because 1 V ≡ 1 J/C, the electric field value 166.7 V/m = 166.7 N/C. Thus, D is the best answer.

An astronaut on Earth notes that in her soft drink an ice cube floats with 9/10 of its volume submerged. If she were instead in a lunar module parked on the Moon where the gravitation force is 1/6 that of Earth, the ice in the same soft drink would float: A. with more than 9/10 submerged. B. with 9/10 submerged. C. with 6/10 submerged. D. totally submerged.

Answer: B Wice = mg = (density of the liquid) × (volume of the object) × (acceleration of gravity) = buoyant force that is pointing upward Note that both the weight and the buoyant force are proportional to g, making the numerical value of g irrelevant to the volume of the ice cube that is submerged. Thus, B is the best answer.

If the speed of the charged particle in an electrical circuit is increased by a factor of 2, the electrical force on the particle will: A. decrease by a factor of 2. B. remain the same. C. increase by a factor of 2. D. increase by a factor of 4.

B. Remain the same Electrical force depends on the particle's charge and the strength of the electric field experienced by the particle, not on the particle's speed.

Destructive interference occurs in photodiode detectors when direct and scattered light rays take paths to the photocell that differ in phase by: A. 0 degrees. B. 90 degrees. C. 180 degrees. D. 360 degrees.

C. Half a wave difference in phase between two waves corresponds to destructive interference

Which circuit elements store energy? I. Capacitors II. Resistors III. Batteries A. I only B. I and II only C. I and III only D. II and III only

C. I and III only

Making which of the following changes to a circuit element will increase the capacitance of the capacitor described in the passage? A. Replacing the 500-Ω resistor with a 250-Ω resistor B. Replacing the 10-V battery with a 20-V battery C. Increasing the separation of the capacitor plates D. Increasing the area of the capacitor plates

D is correct Capacitance C depends on geometric factors only, and in the case of parallel plates, C is proportional to the plate area and inversely proportional to the separation distance of the plates.

In order to determine the relative speed of approach of a sound source by Doppler measurements, three of the following items of data are necessary. Which one is NOT required? A. The speed of sound in the medium B. The frequency of the emitted sound C. The frequency of the observed sound D. The distance between source and observer

D is not necessary Doppler Equation f' = (v+vo) / (v-vs) f where: f' = observed frequency v = velocity of sound waves vo = velocity observer vs = velocity source f = actual frequency

A stationary receiver detects a change in frequency of the signal from a jet flying directly away from it at 300 m/s. Which of the following receivers will detect the same change in frequency from a jet moving away at 600 m/s? A. A receiver moving at 900 m/s in the opposite direction as the jet B. A receiver moving at 300 m/s in the opposite direction as the jet C. A stationary receiver D. A receiver moving at 300 m/s in the same direction as the jet

Doppler effect depends on the relative motion of the source and detector. D is correct

Will light entering a denser medium refract toward or away from the normal?

It will refract TOWARD the normal (smaller angle than the angle of incidence)

Ball 2 is in the water 20 cm above Ball 3. What is the approximate difference in pressure between the 2 balls? A. 2 × 10^2 N/m2 B. 5 × 10^2 N/m2 C. 2 × 10^3 N/m2 D. 5 × 10^3 N/m2

The absolute pressure p at depth h below the surface of a fluid is: p = p0 + ρgh, where p0 is the atmospheric pressure above the liquid, ρ, is the density of the liquid, and g = 9.8 m/s2. Thus, the pressure difference Δp between two locations in the water would be Δp = ρgΔh, where Δh is the difference in depth. Here, Δh = 20 cm = 0.20 m. Thus, Δp = (1000 kg/m3)(9.8 m/s2)(0.20 m) = 1960 Pa = 2.0 x 103 Pa.

The rate at which charge is transferred from one plate to the other is termed: A. current. B. voltage. C. power. D. energy.

The answer is A

If an experiment were conducted using an ideal gas, the work done by the gas in reaching equilibrium would be: A. equal to zero. B. positive. C. negative. D. cannot be determined.

The answer is A ΔE = Q - W E = 0 in ideal gas so W = 0

The 14C atom undergoes beta decay to 14N. What particle is being emitted from the carbon nucleus? A. Electron B. Alpha C. Neutron D. Positron

The answer is A DO NOT ASSUME POSITRON EMISSION! can be both positron and electron! Positron will move left (loss in atomic #), electron will move right (gain in atomic #) on periodic table

Sound of a known frequency, wavelength, intensity, and speed travels through air and bounces off an imperfect reflector which is moving toward the source. Which of the following properties of the sound remains the same before and after reflection? A. Speed B. Intensity C. Frequency D. Wavelength

The answer is A The speed of sound remains CONSTANT in air

A nucleus splits into two fragments that have equal charge but unequal mass. Which of the following is equal for the two fragments as they move apart? A. Magnitude of the force of one fragment on the other B. Magnitude of acceleration C. Speed D. Kinetic energy

The answer is A The unequal-mass fragments move apart under the influence of the mutual Coulomb forces. These forces are equal and opposite (Newton's third law)

Suppose that the railcar passes by a horn that is emitting a sound with frequency f. Which of the following describes the frequency f' that the person on the railcar hears? A. f' > f before passing the horn, f' < f after passing it B. f'< f before passing the horn, f' > f after passing it C. f' = f before passing the horn, f' = f after passing it D. f' > f before passing the horn, f' > f after passing it

The answer is A due to the Doppler effect, the frequency that the person on the railcar hears before passing the horn is larger than the actual frequency of the sound emitted, while the person hears a frequency lower than the actual frequency after passing the horn.

If a modern portable defibrillator uses as 12 V battery and a 20 μF capacitor, what is the total charge stored on the plates of the capacitor? A. 0.24 mC B. 24 mC C. 24 C D. 60 C

The answer is A C = qV 1.2x10^1 V (20x10^-6) = 24x10^-5 = 0.24x10^-3 C

Assume that the side of the water tank is punctured 5.0 m below the top of the water, and that atmospheric pressure is 1.0 × 105 N/m2. What is the approximate speed of the water flowing from the hole? A. 10 m/s B. 12 m/s C. 14 m/s D. 17 m/s

The answer is A. Bernouilles equation states that P2 + ½ ρv^2 + ρgy2 = P1 + ½ ρv^2 + ρgy1. Take point 2 to be at the location of the puncture and point 1 to be at the upper surface of the fluid. At point 1 above the fluid the pressure P1 is the atmospheric pressure. Atm p cancel v1 is negligible density of water is the same = v^2 = [2g(y1 - y2)]. v^2 = [2 (9.8 m/s2) (5.0 m)] = 10 m/s.

Absorption of ultraviolet light by organic molecules always results in what process? A.Bond breaking B.Excitation of bound electrons C.Vibration of atoms in polar bonds D.Ejection of bound electrons

The answer is B

Tritium (H3) differs from typical elemental hydrogen in: A. its number of protons only. B. its number of neutrons only. C. both its number of neutrons and its number of protons. D. neither its number of neutrons nor its number of protons.

The answer is B Isotopes of the same element contain the same number of protons( if not this would change the atomic number and would no longer be hydrogen) but differ numbers of neutrons. Tritium, 3H, has 1 proton and 2 neutrons, while ordinary hydrogen, 1H, has 1 proton and no neutrons.

How much heat is produced from the complete combustion of 30.0 g of methane, if the enthalpy of reaction is -890 kJ/mol? A. 1.7 x 103 J B. 1.7 x 106 J C. 4.7 x 106 J D. 4.7 x 109 J

The answer is B Methane = CH4 = 12 g/mol + 4 g/mol = 16 g/mol 890 KJ/mol / 20 g/mol x 30 g = 1800 KJ = 1.8x10^6 J

When the oxidizer and the fuel react together to produce an explosion, the resulting sound waves are: A. longitudinal, causing air molecules to move perpendicular to the direction of wave propagation. B. longitudinal, causing air molecules to move parallel to the direction of wave propagation. C. transverse, causing air molecules to move perpendicular to the direction of wave propagation. D. transverse, causing air molecules to move parallel to the direction of wave propagation.

The answer is B Sound waves are longitudinal

A mass of 10kg is dropped from 20 m. What is the max speed achieved by this mass? A. 10 m/s B. 20 m/s C. 5 m/s D. 30 m/s

The answer is B The mast first starts out with on PE PE = ugh = 10(10)(20) = 2,000 J Then the PE --> KE KE = 1/2mv^2 2000 = 1/2 (10)(v)^2 2(2000)/10 = v^2 400 = v^2 v = 20 m/s

A ray of light in air is incident upon a glass plate at an angle of 45°. The angle of refraction of the ray in the glass is 30°. What is the index of refraction of the glass? sin 30° = 0.500, sin 45°= 0.707, sin 60° = 0.866, tan 30° = 0.577, tan 45°= 1.000) A. 1.22 B. 1.41 C. 1.57 D. 1.65

The answer is B This is a Snell Law problem: where the n's are the indices of refraction. Taking the index of refraction of air to be nearly 1 then: n1 sin (theta1) = n2 sin (theta2)

Atherosclerosis, a particular type of arteriosclerosis, usually involves either complete blockage or severe restriction of blood flow due to high cholesterol and buildup of plaque in the arteries. In extreme cases, constricted arteries may collapse under pressure from outside the arteries. This can best be explained by: A. an increase in blood viscosity resulting in an extreme increase in blood flow. B. the necessary increase in fluid velocity through the constricted arterial sections causing a corresponding drop in fluid pressure. C. the necessary decrease in fluid velocity through the constricted arterial sections causing a corresponding increase in fluid pressure. D. the increased work required to push through increasing the potential energy density in constricted sections, causing a corresponding pressure drop.

The answer is B When the cross sectional area decreases, velocity increases. When velocity increases, pressure DECREASES

If the magnitude of a positive charge is tripled, what is the ratio of the original value of the electric field at a point to the new value of the electric field at that same point? A 1:2 B. 1:3 C. 1:6 D. 1:9

The answer is B because E = kq/r^2 so when q is triples so is the electric field

A ray of light in air strikes the flat surface of a liquid, resulting in a reflected ray and a refracted ray. If the angle of reflection is known, what additional information is needed in order to determine the relative refractive index of the liquid compared to air? A. Angle of incidence B. Angle of refraction C. Refractive index of air D. Wavelength of the light

The answer is B we know the angle of incidence because its = to the angle of reflection

What is the force acting on a 5 kg object with constant speed? A. 5 N B. 0 N C. 50 N D 10 N

The answer is B. 0 At constant speed the force acting on a n object is 0

Fluid pressure changes with depth are assumed to be linear. Which statements best explains why this does not hold true for atmospheric pressure gases? A. at high temps, gases behave not as ideally B. the volume of mass of air is not constant C. the majority of molecules are non polar D. air is not uniform in composition

The answer is B. The composition of air remains the same, however, air is compressible while liquid is not, meaning their densities change with increasing pressure.

A balloon has a volume of 3.0 L at 25°C. What is the approximate volume of the balloon at 50°C? A. 1.5 L B. 2.0 C. 3.3 L D. 6.0 L

The answer is C

As an ideal fluid flows through a tube of progressively decreasing diameter, the fluid speed: A. decreases. B. decreases and then increases. C. increases. D. remains constant.

The answer is C

If the circuit is closed and a neutral molecule with a dipole moment is oriented between the plates so that the dipole is at a 45° angle to the plates, then the molecule will experience: A. no net force and no net torque. B. a net force but no net torque. C. no net force but a net torque. D. both a net force and a net torque.

The answer is C

A 60-Ω resistor is connected in parallel with a 20-Ω resistor. What is the equivalent resistance of the combination? A.80 Ω B.40 Ω C.15 Ω D.3 Ω

The answer is C 1/60 + 1/20 = 4/60 = 1/15 = 15

In the adiabatic expansion of a real gas, the decrease in temperature is accompanied by: A. an increase in the internal energy of the gas. B. a loss of heat by the gas to the surroundings. C. a decrease in the intermolecular forces of the gas. D. positive work being done on the gas by the environment.

The answer is C A. internal energy decreases with temp B. adiabatic = no loss of heat D. postive for done on the environment not the gas

The frequency used in U/S imaging must be greater than: A. 1 kHz. B. 10 kHz. C. 20 kHz. D. 40 MHz.

The answer is C Ultrasound is defined as sound with a frequency ABOVE the human range of hearing ("ultra" = "above"). To answer this question, then, we need to know what the human range of hearing is. This range is 20 Hz to 20 kHz, so anything greater than 20 kHz qualifies as ultrasound.

Which action requires a larger absolute value of work on a pulley system: lifting the weight from A to B with constant speed, or lowering the weight from B to A with the same constant speed? A. Lifting from A to B B. Lowering from B to A C. Equal absolute value of work in both actions D. No work is required using a pulley.

The answer is C becausee absolute value of the work done is mgΔh and none of the values change.

An artery is constricted at one location to 1/2 its normal cross-sectional area. How does the speed of blood past the constriction compare to the speed of blood flow in the rest of the artery? (Note: Assume ideal fluid flow.) A. It is 1/4 as fast. B. It is 1/2 as fast. C. It is 2 times as fast. D. It is 4 times as fast.

The answer is C if the area is decreased by half then the blood must flow twice as fast to make up for it

Which of the following will bring about an increase in the ratio of separated charge stored on the plates to the potential difference between the plates of a parallel-plate capacitor? I. An increase in the distance between the plates II. An increase in the area of the plates III. An increase in the dielectric constant of the material between the plates A. I and II only B. I and III only C. II and III only D. I, II, and III

The answer is C the equation C = Q/V turns into C = K(ε0A /d) when a dielectric is in place meaning increasing A or K would increase C

In a follow-up experiment, two identical gurneys are placed side-by-side on a ramp with their wheels locked to eliminate spinning. Gurney 1 has a dummy placed on it to give it a total mass of 200 kg, while Gurney 2 is loaded with a dummy that makes it only 50 kg overall. If the ramp has a coefficient of friction of μs, which gurney is more likely to slide down the ramp? A. Gurney 1, due to the increased force of gravity B. Gurney 2, due to the reduced force of static friction C. Gurney 1 and Gurney 2 are equally likely to slide. D. Neither gurney can slide unless the wheels are unlocked.

The answer is C, mass is NOT a factor in whether the gurney will slide down the ramp Gurney one does experience a large force of gravity but this also INCREASES the static friction force proportionally

That the electric field is uniform between the electrodes means that the electric field lines: A.are more closely spaced at the positive electrode than at the negative one. B.intersect halfway between the electrodes. C.are more closely spaced at the negative electrode than at the positive one. D.are equally spaced at both electrodes and between them.

The answer is D

The half-life of a radioactive material is: A.half the time it takes for all of the radioactive nuclei to decay into radioactive nuclei. B.half the time it takes for all of the radioactive nuclei to decay into their daughter nuclei. C.the time it takes for half of all the radioactive nuclei to decay into radioactive nuclei. D.the time it takes for half of all the radioactive nuclei to decay into their daughter nuclei.

The answer is D

The intensity of the radiation emitted by the oxygen sensor is directly proportional to the: A. propagation speed of the radiation. B. wavelength of the radiation. C. polarization of photons emitted. D. number of photons emitted.

The answer is D

Which of the following energy conversions best describes what takes place in a battery-powered resistive circuit when the current is flowing? A.Electric to thermal to chemical B.Chemical to thermal to electric C.Electric to chemical to thermal D.Chemical to electric to thermal

The answer is D

Which of the following particles is expected to have the LEAST mass? A. An alpha particle B. A beta particle C. A positron D. A gamma particle

The answer is D

A glass rod is rubbed with a silk scarf producing a charge of +3.2 × 10-9 C on the rod. (Recall that the magnitude of the proton and electron charges is 1.6 × 10-19 C.) The glass rod has: A.5.1 × 10^11 protons added to it. B.5.1 × 10^11 electrons removed from it. C.2.0 × 10^10 protons added to it. D.2.0 × 10^10 electrons removed from it.

The answer is D 3.2x10^-9/1.6x10^19 You CANT add protons by rubbing the scarf it would require some sort of voltage

An artificial heart valve was tested for its ability to function under extreme conditions, to a maximum flow rate of 4.00 x 10-4 m^3/s. What speed would this correspond to for an average red blood cell within a blood vessel of cross-sectional area 5.00 x 10-6 m^2? A. 2.00 x 10^-9 m/s B. 1.25 x 10^-2 m/s C. 4.05 x 10^-4 m/s D. 8.00 x 10^1 m/s

The answer is D 4.0x10^-4 m^3/s / 5x10^-6 m^2 m^3/m^2 = m 0.8x10^2 m/s = 8x10^1 m/s

What fraction of a 15O sample decays in 10 min if it has a half life of 2 min? A .1/8 B. 9/16 C. 3/4 D. 31/32

The answer is D Because the half-life of 15O is 2 minutes; thus, 10 minutes = 5 half-lives. Therefore, only (1/2)5 = 1/32 of the sample will be left after 10 minutes, while 31/32 of the sample will decay. I got this right but I was confused because I was looking for 1/32 but this is what was LEFT AFTER 31/32 of the sample had decayed.

What is the average power consumed by a 64-year-old woman during the ascent of the 15-cm-high steps, if her mass is 54 kg in 27 s? A.10 W B.20 W C.40 W D.90 W

The answer is D P = W/t = PE/t = mgh/t 54 kg (10 m/s^2) (0.15 m x 30 steps) / 27 s = 90 W

Why must the person sitting in a chair either lean forward or slide their feet under the chair in order to stand up? A.To increase the force required to stand up B.To use the friction with the ground C.To reduce the energy required to stand up D.To keep the body in equilibrium while rising

The answer is D The person is in equilibrium only when the center of mass is directly above their feet. Otherwise, if the person did not lean forward or slide the feet under the chair, the person would fall backward due to the large torque created by the combination of the weight of the body

Which of the following is NOT true of a magnetic field? A. It can be generated by a moving charge. B. It can accelerate a moving charge. C. It can exert a force on a moving charge. D. It can increase the speed of a moving charge.

The answer is D a magnetic field CAN NOT increase the speed of a moving charge because a charged particle is always moving perpendicular to the magnetic field

1H NMR spectroscopy can provide all of the following information EXCEPT: A. the connectivity of atoms in a molecule. B. the number of nonequivalent hydrogens in a molecule. C. the chemical environment of the hydrogen atoms in a molecule. D. the splitting patterns of equivalent hydrogen atoms.

The answer is D splitting patterns provide information of connectivity but they do not apply to equivalent H's, only to NONequivalent H's

The composition of oxygen and nitrogen in air does not change with altitude, the decreasing temperature at high altitude does change the percent of air that is composed of H2O. Assuming constant relative humidity, which of the following is true concerning the total grams of H2O in air at 3000 m above sea level as compared to at sea level? A. Assuming constant relative humidity means that air has roughly the same mass of H2O per unit volume at 3000 m above sea level. B. Whether air at very high altitude has more or less mass of H2O per unit volume than it does at sea level depends on the temperature at high altitude. C. Air has significantly more mass of H2O per unit volume at 3000 m above sea level. D. Air has significantly less mass of H2O per unit volume at 3000 m above sea level.

The answer is D. With decreasing temperature, air is able to hold less H2O. When water freezes it sinks, meaning the higher up, the less water.

A 7-N force and an 11-N force act on an object at the same time. Which of the following CANNOT be the magnitude of the sum of these forces? A. 2 N B. 8 N C. 12 N D. 18 N

The correct answer is A The largest net force occurs when the forces point in the same direction; then, the magnitudes will add mathematically to a value of 18 N. Similarly, the smallest net force occurs when the forces point in opposite directions and the lesser magnitude is subtracted from the greater; this results in a value of 4 N. Of the answers listed, only A (2 N) does not fall between these values.

At a given temperature, the resistance of a wire to direct current depends only on the: A. voltage applied across the wire. B. resistivity, length, and cross-sectional area. C. inductance, length, and cross-sectional area. D. resistivity, length, and capacitance.

The correct answer is B Resistance = Resistivity (L/A)

What happens to the movement of an electron after it is ejected from the cathode in an electric field? A. It is stationary until collisions propel it toward the anode. B. It moves with constant speed toward the anode. C. It accelerates toward the anode. D. It exits through a side of the vacuum photodiode.

The correct answer is C. A charged particle accelerates in an electric field. The electron starts with a velocity that increases as it approaches the anode through the vacuum.

After a block began to slide, how did its speed vary with time? (Note: Assume that the tension and kinetic friction forces on the block were constant in magnitude.) A. It was constant in time. B. It increased exponentially with time. C. It was first constant, then increased linearly with time. D. It increased linearly with time.

The correct answer is D because the block is consistently accelerating! not at constant v due to gravity

What is the wavelength of a laser?

The wavelength of visible light is between about 400 to 750 nm. Red light in particular ranges from 620-750 nm. Helium-neon red lasers operate at 633 nm.

What is the approximate molarity of NaCl in ocean water if 3.5 % of ocean water is NaCl and the density of ocean water is 1.028 kg/L?

There are 1028 g of water / L so, 1028 x 0.035 = 36 g NaCl 36 g NaCl/58 g/mol NaCl = 0.62 mol so there are 0.62 mol/L NaCl

Which of the following electronic transitions for a hydrogen atom would result in the emission of a photon that would be visible to the human eye? A. n = 1 to n = 3 B. n = 2 to n = 4 C. n = 2 to n = 1 D. n = 4 to n = 2

Think about it this way: the visible portion is between 400 and 800 nm, so we are looking for a value that will lie between them. Since the Rydberg constant is basically 107, we can ignore it for now. So we have an equation that looks like: 1/wavelength = 1/final2 - 1/initial2 Substituting for C and D gives us: 1/wavelengthC = 1/1 - 1/4 => wavelengthC = 4/3 1/wavelengthD = 1/4 - 1/16 => wavelengthD = 16/3 so C = ~1 and D = ~5 500 falls between 400-800 of visible light

coordinate covalent bond

a covalent bond in which one atom contributes both bonding electrons

converging lens

a lens that is thickest in the middle causing parallel light rays to come together at a point Form real inverted images

An ultrasound examination could show the motion of a fetus. In order to image this motion, the ultrasound examination devices requires what minimal information? A. The speeds of the sound and of the moving object. B. The speed of the sound, and the frequencies of the sound waves emitted and observed. C. The speeds of the sound and of the moving object, and the frequencies of the sound waves emitted and observed. D. The speeds of the sound and of the moving object, and the frequencies and wavelengths of the sound waves emitted and observed.

the answer is B The purpose of the probe is to see motion in the fetus, so knowing the speed of the moving object beforehand is unrealistic: that's what it is trying to find! This eliminates choices A, C, and D. Choice B is correct because the Doppler shift relates frequencies at the source and the detector (which was the original source prior to frequency shifting upon reflection off of the fetus), the speed of sound, and speeds of the source and detector (in this case, the same object, again due to reflection). Knowing the first three of these determines the fourth.

The advantage of the Doppler ultrasound technique over the standard ultrasound technique is that it also allows: A. distinguishing between fluids and tissue. B. measuring the blood flow. C. measuring the tissue density. D. measuring the heart wall thickness.

the answer is B sonic Doppler shift is useful for determining relative motion of the source and detector of a sound wave. Blood flow is moving. a traditional ultrasound can do all of the other options as well

Myopia is a condition of the eye where the crystalline lens focuses the light rays to a position between the lens and the retina. To correct for this condition, a thin lens is placed in front of the eye that will help to focus light on the retina. The type of lens required is: A. cylindrical. B. converging. C. diverging. D. spherical.

the answer is C myopic eye focuses light from distant objects prior to its reaching the retina, and the addition of a diverging lens moves the focal point further back to reach the retina A cylindrical lens is used to correct ASTIGMATISM

What information about an axon is required to calculate the current associated with an NCV pulse? A. Conductivity, resistivity, and length B. Potential, conductivity, and radius C. Potential, resistivity, and radius D. Potential, resistance per unit length, and length

the answer is D ohm's Law, I = V/R.

A 0.5-kg uniform meter stick is suspended by a single string at the 30-cm mark. A 0.2-kg mass hangs at the 80 cm mark. What mass hung at the 10-cm mark will produce equilibrium? A. 0.3 kg B. 0.5 kg C. 0.7 kg D. 1.0 kg

the correct answer is D At equilibrium the net torque left of the suspension must equal the net torque to the right of it. The torque of force mg applied at a distance d from the suspension is mgdsinθ, with θ is the angle between the stick and the force. At equilibrium the forces are all vertical while the beam is horizontal, hence θ = 90° and sin90° = 1. Thus, the torque left of the suspension will be mg(20 cm). There are two torques right of the suspension, the hanging mass torque (0.2 kg)g(50 cm) and the weight of the meter stick's torque (0.5 kg)g(20 cm), where 20 cm represents the distance from the suspension to the center of mass of the meter stick. Thus, the net right torque is (0.2 kg)g(50 cm) + (0.5 kg)g(20 cm). Equating the left and right net torques yields m = 1 kg.

Diverging lenses (concave)

these lenses have a negative focal length which means they are diverging lenses. Such lenses form virtual and reduced images of objects situated at distances larger than the focal length.


संबंधित स्टडी सेट्स

Chapter 2: Theory, Research, and Evidence-Informed Practice

View Set

Module 2: Transformations and congruence

View Set

Nursing Application for Bowel Elimination

View Set

Chapters 11 and 12 Quiz Optional

View Set

Chemistry 116: Exam #1 Study Guide

View Set

Periodic Table of Elements 16-30

View Set